Đến nội dung

vietfrog nội dung

Có 829 mục bởi vietfrog (Tìm giới hạn từ 28-04-2020)



Sắp theo                Sắp xếp  

#280179 Ảnh thành viên

Đã gửi bởi vietfrog on 25-10-2011 - 23:33 trong Góc giao lưu

Hôm nay up ảnh chụp chung với mấy đứa bạn cho mọi người cùng đánh giá :wub:

Mình ngồi góc ngoài cùng bên phải :closedeyes:


Kaka. Cuối cùng cũng thấy cậu Phúc.
Trông lớn ghê. Gặp ngoài đường chắc mình chào bằng anh mất :icon6:



#279199 Ảnh thành viên

Đã gửi bởi vietfrog on 16-10-2011 - 15:56 trong Góc giao lưu

@to Khánh:Cậu Khánh sang I-Rắc chơi à >:) >:) .

@to Giang1994: Cậu còn định gạ gẫm mình post ảnh người yêu lên làm gì :tongue: , người yêu mình học chuyên Văn, không phù hợp với VMF ta. :icon6: :icon6: :icon6: :icon6: .

@to Phúc: Bác Phúc post ảnh mau lẹ lên nhé, hội đồng nhiếp ảnh đang rất hồi hộp đấy. :icon6: :icon6:



#279130 Ảnh thành viên

Đã gửi bởi vietfrog on 16-10-2011 - 00:53 trong Góc giao lưu

Ec ec. Nhiều anh em đưa ảnh lên quá. :tongue: .Tự tin phát.
Ảnh của mình đây. Đề nghị ông Phúc (dark_templar ) bắt trước tôi ngay. :ukliam2: :ukliam2:
Hình đã gửi



#283608 Ảnh thành viên

Đã gửi bởi vietfrog on 15-11-2011 - 21:52 trong Góc giao lưu

Hix, lục lọi lại mới tìm thấy 1 cái trên Zing me @@

Anh vietfrog đẹp trai nhỉ ^^

Chú cứ quá khen. Anh bình thường thôi mà :wub: :wub: :wub: :wub:



#286562 Ảnh thành viên

Đã gửi bởi vietfrog on 04-12-2011 - 19:00 trong Góc giao lưu

Cắm trại hả anh?



#288078 Ảnh thành viên

Đã gửi bởi vietfrog on 13-12-2011 - 21:33 trong Góc giao lưu

Vậy thì anh Thế làm Bang chủ Cái bang rồi :))



#288066 Ảnh thành viên

Đã gửi bởi vietfrog on 13-12-2011 - 21:13 trong Góc giao lưu

Kaka. :icon6: Ca sĩ này là ai?? Ngọc Thế chứ ai :icon6:



#279105 Ảnh thành viên

Đã gửi bởi vietfrog on 15-10-2011 - 21:28 trong Góc giao lưu

Cậu Giang với Toàn đẹp zai ghê.
-Mình thực sự bất ngờ về Giang. Trông rất Giang.... giang hồ. :tongue: :tongue: .
-Còn Toàn, lớp 8 gì mà to cao thế kia? :ohmy: .''Ăn gì to béo đẫy đà làm sao'' :tongue: .

Tôi đề nghị anh Dark_templar ( Phúc) đưa ảnh lên ngay, tôi thách anh đấy! :angry: :angry: :angry: :angry:



#279342 Ảnh thành viên

Đã gửi bởi vietfrog on 17-10-2011 - 21:57 trong Góc giao lưu


Vietfrog có người yêu là dân chuyên Văn cơ à :D. Hơn anh rồi đó nha, anh tủi quá :( :( :( .

Anh Thành post ảnh anh lên cho bọn em chấm điểm đê.
Khó khăn mỗi ảnh của anh với ảnh của cậu dark_templar thui đó.
Anh post ngay nha.
Mà anh tủi gì mà tủi? Chắc người yêu anh chuyên Toán nốt hả :icon6: . Được cả đôi!



#272346 Topic về Bất đẳng thức, cực trị THCS

Đã gửi bởi vietfrog on 13-08-2011 - 23:43 trong Bất đẳng thức và cực trị

Hỏi nhiều bài vậy em. Nhiều bài tương tự những bài em vừa hỏi rồi mà.
Em phải học gõ Latex đi nhé, không lần sau anh del bài đó!
Gợi ý:
Bài 1:
Chứng minh:
$\dfrac{a}{{a + b + c + d}} < \dfrac{a}{{b + c + a}} < \dfrac{{a + d}}{{a + b + c + d}}$
Tương tự sau đó công lại thấy $1<A<2$ suy ra A không nguyên!
Bài 2:
Áp dụng BĐT Shwarz có ngay:

$\dfrac{{{a^2}}}{{b + c}} + \dfrac{{{b^2}}}{{a + c}} + \dfrac{{{c^2}}}{{b + a}} \ge \dfrac{{{{(a + b + c)}^2}}}{{2(a + b + c)}} = 1$

Bài 3:
Thế ${a^2} + {b^2} + {c^2} = 1$ vào và biến đổi là được.
Bài 4:
Giả thiết tương đương:
$ab + bc + ac = 0$
Bài 5:
....



#295404 Topic bất đẳng thức THCS (2)

Đã gửi bởi vietfrog on 22-01-2012 - 19:05 trong Bất đẳng thức và cực trị

Bài 153:Cho tam giác có độ dài 3 cạnh là a,b,c chu vi bằng 2. Tìm GTNN của
$P=4(a^3+b^3+c^3)+15abc$

Giả thiết tương đương : $a+b+c=4$.
Sau đó dùng phương pháp Look at the end point là xong. Khá ngắn gọn :D
Có thể tham khảo phương pháp tại đây!



#292584 Topic bất đẳng thức THCS (2)

Đã gửi bởi vietfrog on 06-01-2012 - 22:08 trong Bất đẳng thức và cực trị

Góp vào Topic của Kiên một bài:

Bài 32 :
Cho $a,b,c$ là 3 số thực dương thỏa $a,b,c < 4$.
Chứng minh bất đẳng thức: $$\dfrac{1}{4-a}+\dfrac{1}{4-b}+\dfrac{1}{4-c}\geq \dfrac{3}{4}+\dfrac{a^2+b^2+c^2}{16}$$



#296510 Topic bất đẳng thức THCS (2)

Đã gửi bởi vietfrog on 26-01-2012 - 11:05 trong Bất đẳng thức và cực trị

Anh xin lỗi do không theo dõi từ đầu nên post lặp xin thay bằng bài khác (Không biết có lặp nữa không)
Bài 202:Cho các số thực dương $a,b,c$ có tổng bằng $3$.Chứng minh rằng
$$3({a^2} + {b^2} + {c^2}) + 4abc \ge 13$$

Bài này không cần là $3$ cạnh tam giác đâu.
Lời giải ( Phương pháp Look at the end point )
Ta có: \[\begin{array}{l}
A = 3\left( {{a^2} + {b^2} + {c^2}} \right) + 4abc - 13 \\
\Leftrightarrow A = 3\left( {{{\left( {a + b} \right)}^2} - 2ab} \right) + 3{c^2} + 4abc - 13 \\
\Leftrightarrow A = 3{\left( {3 - c} \right)^2} - 6ab + 3{c^2} + 4abc - 13 \\
\Leftrightarrow A = ab\left( {4c - 6} \right) + 6{c^2} - 18c + 14 \\
\end{array}\]
Bằng BĐT $AM-GM$ ta dễ dàng có được: $ab \in \left[ {0;\frac{{{{\left( {3 - c} \right)}^2}}}{4}} \right]$
Xét:$A = f(ab) = ab\left( {4c - 6} \right) + 6{c^2} - 18c + 14\backslash \left[ {0;\frac{{{{\left( {3 - c} \right)}^2}}}{4}} \right]$
Đây là một hàm bậc nhất ẩn $ab$. Ta luôn có: $Minf(ab) = Min\left\{ {f(0);f\left( {\frac{{{{(c - 3)}^2}}}{4}} \right)} \right\}$
Ta chứng minh: $\left\{ \begin{array}{l}
f(0) \ge 0 \\
f\left( {\frac{{{{(c - 3)}^2}}}{4}} \right) \ge 0 \\
\end{array} \right.$
Thật vậy: \[\left\{ \begin{array}{l}
f\left( 0 \right) = 6{c^2} - 18c + 14 > 0 \\
f\left( {\frac{{{{(c - 3)}^2}}}{4}} \right) = {\left( {c - 1} \right)^2}\left( {c + \frac{1}{2}} \right) \ge 0 \\
\end{array} \right.\]
Như vậy thì : \[A \ge 0 \Leftrightarrow 3\left( {{a^2} + {b^2} + {c^2}} \right) + 4abc - 13 \ge 0\]
Ta có được đpcm.
Dấu ''='' khi $a=b=c=1$.
------------------------------------------
P/s: Các em THCS hoàn toàn có thể sử dụng. Tuy đây là cách không ngắn gọn nhưng có thể áp dụng nhiều bài. :D
Đề lại một bài nhé.
Bài 203:
Cho $a,b,c$ là 3 số dương có tổng bằng 3. Tìm GTLN,GTNN nếu có của:

\[A = 2\left( {{a^3} + {b^3} + {c^3}} \right) + 3\left( {ab + ac + bc} \right) + 5abc\]



#302285 Topic bất đẳng thức THCS (2)

Đã gửi bởi vietfrog on 05-03-2012 - 00:16 trong Bất đẳng thức và cực trị

Bài anh Thành có xảy ra dấu = đâu?
Với cách chứng minh đó thì dấu bằng xảy ra tại $x;y;z \in \left\{ {1;3} \right\}$.
Dễ thấy không có bộ số nào thỏa mãn.
Theo anh, cách chứng đó phù hợp với bài của Phúc. :D
$x=y=1;z=3$ hoặc $x=y=3;z=1$ ra VP của Phúc :D



#295350 VMF NEXT TOP MODEL - Thảo luận - Bình "loạn"

Đã gửi bởi vietfrog on 22-01-2012 - 16:35 trong Góc giao lưu

Cứ đà này thì Hoàng đoạt giải rồi :(. Mình tham gia muộn quá :D .



#295101 Tản mạn BĐT

Đã gửi bởi vietfrog on 21-01-2012 - 21:33 trong Bất đẳng thức và cực trị

Post một số bài nên cho anh em chém nhá. Cảm ơn mọi người đã tham gia Topic trong thời gian qua.
Bài 116:
Cho các số $a,b,c$ dương thỏa mãn: $\left\{ \begin{array}{l}
a + b + c = 6 \\
{a^2} + {b^2} + {c^2} = 14 \\
\end{array} \right.$
Tìm GTLN,GTNN của $P = \frac{{4a + b}}{c}$
Bài 117:
Chứng minh rằng:

\[\frac{{\sin x + 2\cos \frac{x}{2}}}{{\cos x + 2\sin \frac{x}{2}}} \ge 1 + \frac{{\sqrt 2 }}{2},\forall x \in \left[ {0;\frac{\pi }{2}} \right]\]

Bài 118:
Cho $2x-y=2$. Chứng minh rằng:

\[\sqrt {{x^2} + {{\left( {y + 1} \right)}^2}} + \sqrt {{x^2} + {{\left( {y - 3} \right)}^2}} \]

Bài 119:
Cho $x,y \ge 0;{x^3} + {y^3} = 1$
Tìm giá trị lớn nhất của: \[P = \sqrt x + 2\sqrt y \]



#313988 Tản mạn BĐT

Đã gửi bởi vietfrog on 02-05-2012 - 20:53 trong Bất đẳng thức và cực trị

Lời giải :
Ta dễ dàng nhận ra , đây là bài tìm giá trị nhỏ nhất cua ba bất đẳng thức nhỏ :

$A = \frac{a}{b+c}+\frac{b}{a+c}+\frac{c}{a+b} \geq \frac{3}{2} ( Nesbit )$
$B=\frac{b^2}{b+c}+\frac{c^2}{c+a}+\frac{a^2}{a+b} \geq \frac{a+b+c}{2} =\frac{1}{2}$
$C=\frac{c^3}{b+c}+\frac{a^3}{c+a}+\frac{b^3}{a+b} \geq \frac{(a^2+b^2+c^2)^2}{bc+c^2+ac+a^2+ab+b^2}$
$\geq \frac{(a^2+b^2+c^2)^2}{2(a^2+b^2+c^2)}=\frac{a^2+b^2+c^2}{2}$ $\geq \frac{a+b+c}{6}=\frac{1}{6}$
$A+B+C=\frac{3}{2}+\frac{1}{2}+\frac{1}{6}=\frac{13}{6}$
$\Longrightarrow$ điều phải chứng minh

$a,b,c$ đã dương đâu mà áp dụng như vậy. Xem lại nhé! :D



#270546 Tản mạn BĐT

Đã gửi bởi vietfrog on 02-08-2011 - 11:32 trong Bất đẳng thức và cực trị

Bài tiếp theo.
Cho $a_1 ,a_2 ,...,a_n > 0;\,\,\,\,\,S = a_1 + a_2 + \cdot \cdot \cdot + a_n $. Chứng minh rằng:

$\dfrac{{\sqrt {a_1 } }}{{\sqrt {S - a_1 } }} + \dfrac{{\sqrt {a_2 } }}{{\sqrt {S - a_2 } }} + \cdot \cdot \cdot + \dfrac{{\sqrt {a_n } }}{{\sqrt {S - a_n } }}\,\,\,\,\, \ge \,\,\,\,2;\,\,\,\,\left( {n \ge 2} \right)$.

------------------------

KHÔNG THỬ SAO BIẾT!!!

Bất đẳng thức đã cho tương đương:

$\sum\limits_{i = 1}^n {\sqrt {\dfrac{{{a_i}}}{{S - {a_i}}}} } \ge 2$

Ta đánh giá:

$\sqrt {\dfrac{{{a_i}}}{{S - {a_i}}}} = \sqrt {\dfrac{{a_i^2}}{{(S - {a_i}).{a_i}}}} = \dfrac{{{a_i}}}{{\sqrt {(S - {a_i}).{a_i}} }} \ge \dfrac{{{a_i}}}{{\dfrac{S}{2}}} = \dfrac{{2{a_i}}}{S}$

Từ đó suy ra:

$\sum\limits_{i = 1}^n {\sqrt {\dfrac{{{a_i}}}{{S - {a_i}}}} } \ge \dfrac{{2.\sum\limits_{i = 1}^n {{a_i}} }}{S} = 2$

Dấu ''='' xảy ra khi: $n=2;{x_1} = {x_2} > 0$
..........................................



#274539 Tản mạn BĐT

Đã gửi bởi vietfrog on 30-08-2011 - 23:48 trong Bất đẳng thức và cực trị

Bài 47 (Một BĐT nhẹ nhàng)
Cho $a;b;c;x;y;z \ge 0$ thỏa mãn: $a + x = b + y = c + z$
Chứng minh rằng:
$ax + by + cz \le 1$



#275735 Tản mạn BĐT

Đã gửi bởi vietfrog on 08-09-2011 - 22:45 trong Bất đẳng thức và cực trị

Tiện đây post thêm một số BĐT đối xứng đơn giản để mọi người chém nhiều cách.
Bài 53: ( HSG tỉnh Hải Dương vòng 2 năm 2009-2010 )
Cho $a,b,c$ là các số dương và $abc=1$.
Chứng minh rằng:

$\dfrac{a}{{a + {b^4} + {c^4}}} + \dfrac{b}{{b + {c^4} + {a^4}}} + \dfrac{c}{{c + {a^4} + {b^4}}} \le 1$






Bài 54:
Cho các số dương a,b,c thỏa mãn : ${x^2} + {y^2} + {z^2} = 1$
Chứng minh rằng:

$\dfrac{x}{{1 - {x^2}}} + \dfrac{y}{{1 - {y^2}}} + \dfrac{z}{{1 - {z^2}}} \ge \dfrac{{3\sqrt 3 }}{2}$




#267779 Tản mạn BĐT

Đã gửi bởi vietfrog on 07-07-2011 - 11:08 trong Bất đẳng thức và cực trị

Tặng các bạn 1 bài khá đẹp :
Cho $ a,b,c $ là 3 cạnh của 1 tam giác . Chứng minh rằng :
$ (ab)^2+(bc)^2+(ca)^2 \leq a^3b+b^3c+c^3a $

Giả sử $a \ge b \ge c$

Sử dụng BĐT Chebyshev cho 2 bộ 3 số :
${a^2} + bc \ge {b^2} + ac \ge {c^2} + ab$ và

$\dfrac{1}{a} \le \dfrac{1}{b} \le \dfrac{1}{c}$

Theo BĐT Chebyshev ta có:

$3\sum {\dfrac{{{a^2} + bc}}{a} \le \sum {({a^2} + bc)\sum {\dfrac{1}{a}} } } $

$ \Leftrightarrow 3\sum {(a + \dfrac{{bc}}{a}) \le \sum {(a + b + c + \dfrac{{{a^2}}}{b} + \dfrac{{{a^2}}}{c} + \dfrac{{bc}}{a}} )} $

$ \Leftrightarrow 2\sum {\dfrac{{bc}}{a} \le \sum {(\dfrac{{{a^2}}}{b} + \dfrac{{{a^2}}}{c}) \le 2\sum {\dfrac{{{a^2}}}{c}} } } $

$ \Leftrightarrow \sum {\dfrac{{bc}}{a} \le } \sum {\dfrac{{{a^2}}}{c}} $

$ \Leftrightarrow \sum {{{(bc)}^2} \le \sum {{a^3}b} ({\rm{ ok)}}} $

Hí. Bài nữa đi Lâm ơi!



#273608 Tản mạn BĐT

Đã gửi bởi vietfrog on 22-08-2011 - 22:47 trong Bất đẳng thức và cực trị

Tiếp theo. Chúng ta sẽ thảo luận về BĐT AM-GM với bộ 3 số.

Bài 42
Cho $a,b,c \ge 0$. Chứng minh rằng :

$a + b + c \ge 3\sqrt[3]{{abc}}$



Ta có thể chứng minh BĐT thông dụng trên bằng bao nhiêu cách ???
Hãy trình bày cẩn thận, rõ ràng cách chứng minh của các bạn.
Hy vọng Topic sẽ sôi động trở lại!



#276334 Tản mạn BĐT

Đã gửi bởi vietfrog on 13-09-2011 - 12:24 trong Bất đẳng thức và cực trị

vietfrog thông cảm nha :-? Đang cơn lười gõ Latex ;) Cjhi3 gợi ý thôi -_-

Xài BDT Bernoulli :Leftrightarrow


Bài APMO 2004 có rất nhiều cách giải và đã được mở rộng, làm chặt rất nhiều. Bên cạnh cách như của bạn Hà Quốc Đạt hay cách xài BĐT Bernoulli như của dark templar thì còn có thể có thêm các cách khác.
Không nêu ra hết được nhưng có thể kể tới :
Cách 1: Sử dụng Cauchy-Schwarz :
$({a^2} + 2)({b^2} + c) = ({a^2} + 1)({b^2} + 1) + {a^2} + {b^2} + 3 \ge \dfrac{3}{2}\left[ {{{(a + b)}^2} + 2} \right]$...

Cách 2: Dồn biến với việc đặt : $\dfrac{{a + b}}{2} = t$ sau đó chứng minh : $f(t;t;c) \ge 0$...

@vietfrog :Có rất nhiều hương giải quyết nhưng mong mọi người post lời giải cụ thể lên để các thành viên tiện theo dõi.

@to dark templar : Đi cai bệnh lười gõ latex đi, chứ cứ để đang cơn lười gõ latex thế nguy hiểm lắm -_- . Đi cai đi. VMF vẫn mở rộng vòng tay với những người biết làm lại :Leftrightarrow :Leftrightarrow :Leftrightarrow :Rightarrow



#272920 Tản mạn BĐT

Đã gửi bởi vietfrog on 18-08-2011 - 11:20 trong Bất đẳng thức và cực trị

Bài viết bắt đầu lẻ tẻ và khá khó r�ồi đây.
Còn 3 bài chưa giải quyết triệt để. Mọi người cùng suy nghĩ nhé!
Bài 34. Cho $a,b,c >0$ và $a+b+c = \dfrac{1}{a} + \dfrac{1}{b} + \dfrac{1}{c}$.

Chứng minh rằng $(a+b-c)(b+c-a)(c+a-b) \le 1$

Bài 38
Cho các số thực dương a,b,c .CMR:

$ (a+\dfrac{b^2}{c})^2 + (b+\dfrac{c^2}{a})^2+(c+\dfrac{a^2}{b})^2 \geq 12\dfrac{a^3+b^3+c^3}{a+b+c}$

Bài 39: Cho $a,b,c$ là các số thực dương. Chứng minh rằng:

$\sqrt{a^{4}+\dfrac{b^{4}}{2}+\dfrac{c^{4}}{2}}+\sqrt{b^{4}+\dfrac{c^{4}}{2}+\dfrac{a^{4}}{2}}+\sqrt{c^{4}+\dfrac{a^{4}}{2}+\dfrac{b^{4}}{2}}\geq \sqrt{a^{4}+b^{3}c}+\sqrt{b^{4}+c^{3}a}+\sqrt{c^{4}+a^{3}b}$

P/s: Mọi người hãy post lên cả những ý tưởng nhé!



#270455 Tản mạn BĐT

Đã gửi bởi vietfrog on 01-08-2011 - 14:57 trong Bất đẳng thức và cực trị

Nếu làm giống như bạn thì phải ra $\max f(x)=\dfrac{1}{3}$ mới đúng.
Mà làm như bạn cũng sai rồi.Với $n=2$ thì $\max f(x)=\dfrac{4}{27}$ khi $(x;y;z)=\left(0;\dfrac{2}{3};\dfrac{1}{3} \right)$ chứ không phải là $\dfrac{1}{3}$.:D

Ừ. Mình biết sai rồi :D, mình biết mình làm như thế là không đúng :D, mong mọi người tha thứ cho mình :D:P .
Vậy là còn 2 bài trong topic chưa giải:
Bài 1:

Tổng quát:Cho $n$ số thực dương $a_1;a_2;...;:D$.Chứng minh rằng:

$\dfrac{1}{n} \sum\limits_{i=1}^{n}a_{i} -\sqrt[n]{\prod\limits_{i=1}^{n}a_{i}} \le \max \{(\sqrt{a_{i}}-\sqrt{a_{j}})^2 \};1 \le i \neq j \le n.$

P/s: Có khi bài này dark templar phải làm thui :D
Bài 2:

Cho x, y, z là các số thực không âm và $x + y + z = 1$. Tìm GTLN của hàm số $f(x,y,z) = x^n y + y^n z + z^n x\,\,,\,n \in N$.